Oscilador armónico impulsado con fuerza térmica de Langevin. ¿Cómo extraer la temperatura de x(t)x(t)x(t)?

Supongamos que ha impulsado un oscilador armónico (parámetros: masa, gamma, omega0) mediante una fuerza determinista Fdrive (por ejemplo, una onda sinusoidal). Ahora suponga que agrega la fuerza de Langevin estocástica FL que está relacionada con la temperatura del baño T.

La pregunta es cómo extraer la información sobre la temperatura T observando la traza de tiempo de x(t) observándola durante un tiempo MUCHO MENOR QUE 1/gamma.

Entonces solo puede mirar x(t) una fracción de 1/gamma y quiere saber la temperatura del baño. Ya conoces omega0, gamma y masa.

Creo que es posible, pero no puedo probarlo.

NB: omega0 es la frecuencia de resonancia del oscilador gamma es la tasa de amortiguamiento FL se define como =2gammakBTdeltadirac(t2-t1) y =0

Mire quizás la difusividad de partículas D , que debe estar i) relacionado con la varianza de posición como X 2 ¯ = 2 D t , y ii) proporcional a la temperatura, D k b T . Véase, por ejemplo, web2.clarkson.edu/projects/crcd/me537/downloads/2_Brownian.pdf
En un segundo pensamiento X 2 ¯ = 2 D t en general t , pero aún podrías mirar X 2 ¯ ( t ) y sus derivados incluso en escalas de tiempo más cortas. La proporcionalidad a la temperatura a través de la intensidad de la fuerza de Langevin aún debería estar allí.

Respuestas (1)

Tomando

metro d 2 X d t 2 = k X γ v + F ( t ) + η
y escribiendo esto como
d X t = A X t d t + F t d t + σ d W t
dónde X t = ( X , v ) T , A = ( 0 1 k metro γ metro ) , F t = ( 0 , F ( t ) ) T , σ = ( 0 , 2 γ k B T / metro ) T .

Resolviendo esto, como siempre,

X t = mi t A X 0 + 0 t mi ( s t ) A F s d s + 0 t mi ( s t ) A σ d W s

La solución general aquí es un poco complicada gracias a la matriz exponencial, pero si establece k = 0 todo se simplifica mucho y recuperas el proceso de Ornstein-Uhlenbeck.

Ahora no tengo pruebas para esto (supongo que al menos en condiciones típicas el proceso integrado 0 t 0 t F ( s , t ) d W s d t tiene una varianza menor que 0 t F ( s , t ) d W s , que creo que es equivalente a la declaración ( F ( s , t ) ) 2 > ( s t F ( s , t ) d t ) 2 ), pero probando con simulaciones parecía bastante difícil recuperar la temperatura de la varianza de X t : calculé X t + Δ t dado X t usando la fórmula anterior, luego tomó la varianza de la diferencia de la así predicha X t + Δ t contra lo real X t + Δ t . Esto todavía dejó un término residual debido a la fuerza externa, quizás debido al ruido numérico (en el sentido de que Euler-Maruyama, el método que usé, no coincide numéricamente hablando con la forma en que calculé las integrales con suficiente precisión). Todo esto es para decir que este enfoque es bastante sensible al ruido. Sin embargo, funcionó mucho mejor para la velocidad (nuevamente, ya que su varianza es mayor),

Var ( v t + Δ t v t ) = 0 Δ t ( ( 0 , 1 ) mi ( s Δ t ) A σ ) 2 d s

que como se puede ver depende linealmente de T .

Si no necesita un proceso muy automatizado para hacer esto, probablemente pueda deshacerse de los residuos de una manera más manual.

Gracias por tu respuesta. ¿Podrías explicar el último paso? Parece que Var diverge con Deltat. ¿Es normal?
¿No se puede hacer esto a la manera de Langevin, es decir, escribir la ecuación de movimiento en el dominio de la frecuencia y resolver para X 2 ?
@alarge Sí, según mis simulaciones, sus variaciones son realmente sensibles al ruido. Parece que no hay comida gratis, si quieres saber la energía del oscilador (energía térmica), tienes que esperar... De hecho, si miras el espectro de <x²>, verás un lorentziano de ancho
γ
en frecuencia
ω 0
. Si desea resolver este lorentziano, debe esperar al menos unos pocos \gamma. La integral debajo de tu lorentziano te dará con seguridad la temperatura. La esencia de mi pregunta era "¿podría darse el lujo de no esperar unos
γ
. Parece que no, pero por qué...
@pierebean ¿Puede agregar algunos valores específicos de las variables y veré si tengo mejor suerte al inferir la temperatura?
omega0=2*pi*1e9; gammaM=2*pi*1e9/1e3; (factor Q 1000) meff=9,025e-15; k=omega0^2*meff; Deltat=0,0001*gammaM;
@pierebean Creo que la dificultad en la estimación podría deberse al hecho de que si solo obtenemos la información posicional de la partícula, el estado está incompleto. Si no dejara caer muestras fuera de la ruta simulada (que genero con Euler), la reconstrucción de la velocidad es perfecta y la predicción de la temperatura está dentro de unos pocos puntos porcentuales más o menos. Sin embargo, si simulo más densamente de lo que muestreo, ya no puedo obtener el estado completo y la var se desviará en un 30% más o menos. Creo que podríamos hacerlo mejor si tuviéramos que usar algo más inteligente que una diferencia directa.